Search found 164 matches


Hi! I hope I understand your question correctly. Look at our series: 5,5,5,5,5,6,6,6,6,6,6,7,7,7,7,7,7,7.... an There are 1000 terms in the series. the 5 first terms all are equal to 5 the 6 next terms all are equal to 6 the 7 next terms all are equal to 7 we are looking for an: 5 - 14 5 + 6 + 7 + 8...

by Maciek

Thu Sep 16, 2010 10:25 am
Forum: Problem Solving
Topic: Number Systems (Arthimetic Progression)
Replies: 4
Views: 1577

Hi! This question is also included in "300+ GMAT Math questions with Best Solutions". The explanation is following: Here, we have 4 digits (positive integer less than 10000 means that 9999 is the biggest and we can pretend that "5" is "0005") that must sum to 5. Since w...

by Maciek

Thu Sep 16, 2010 9:23 am
Forum: Problem Solving
Topic: the sum of the digits equals 5
Replies: 6
Views: 7054

Hi! This question is tricky. Not only does it test your knowledge about usage of paired conjunction 'not only ... but also', but it also tests your knowledge about parallelism, modifiers, and logic of the sentence. (A) "did... create... and gave" verbs are not parallel (B) modifier error (...

by Maciek

Thu Sep 16, 2010 9:05 am
Forum: Sentence Correction
Topic: Not only..but also construction question. Source: GMAT Prep
Replies: 5
Views: 3120
by Maciek

Thu Sep 16, 2010 2:22 am
Forum: Sentence Correction
Topic: Few/A few/ little/ a little problem
Replies: 5
Views: 1705

We can't use past perfect here because events occur during the same time period. So, they must be in the same tense.

by Maciek

Thu Sep 16, 2010 2:18 am
Forum: Sentence Correction
Topic: Modifier & tense problem
Replies: 5
Views: 1451

Hi! There are few issues here: quantity adjectives countable and uncountable nouns, and subject-verb agreement 1) quantity adjectives with countable and uncountable nouns We use 'little' with uncountable nouns and 'water' is uncountable noun. We use 'few' with countable plural nouns and 'services' i...

by Maciek

Thu Sep 16, 2010 1:50 am
Forum: Sentence Correction
Topic: Few/A few/ little/ a little problem
Replies: 5
Views: 1705

Hi! IMO E This answer doesn't include verb tense error. Read "Top 10 Tips for the Sentence Correction Section" by Chris Black http://www.beatthegmat.com/mba/2010/09/08/top-10-tips-for-the-sentence-correction-section Read more about modifiers http://www.studygs.net/modcom.htm Hope it helps!...

by Maciek

Thu Sep 16, 2010 1:19 am
Forum: Sentence Correction
Topic: Modifier & tense problem
Replies: 5
Views: 1451

Hi all! IMO E There are following issues here: idiom, modifier, and logical structure 1) idiom correct idiom is "as ... as" According to dictionary.reference.com, this expression is "used to express similarity or equality in a specified characteristic, condition, etc., as between one ...

by Maciek

Wed Sep 15, 2010 1:57 am
Forum: Sentence Correction
Topic: kneton sc 4 as 5
Replies: 2
Views: 1596

Hi! IMO D There are 3 issues here: appropriate verb tense, parallelism, and pronoun-antecedent agreement. 1) appropriate verb tense We use the past perfect for an action which finished in the past and whose result was visible in the past. Therefore, past perfect is appropriate here. There is 3/2 spl...

by Maciek

Wed Sep 15, 2010 1:13 am
Forum: Sentence Correction
Topic: knewton-sc 4 v4
Replies: 3
Views: 1401

It is great progress.
Congratulations!
I hope your first application will be successful.

Best,
Maciek

by Maciek

Tue Sep 14, 2010 10:51 am
Forum: I just Beat The GMAT!
Topic: 490->710 (Q48, V40) Some useful tips.
Replies: 19
Views: 9568

Thanks for clarifying Singhsa! Check this: Breaking Down GMATPrep Weighted Average Problems (Part 1 of 2) http://www.beatthegmat.com/mba/2009/12/13/breaking-down-gmatprep-weighted-average-problems-part-1-of-2 Breaking Down GMATPrep Weighted Average Problems (Part 2 of 2) http://www.beatthegmat.com/m...

by Maciek

Tue Sep 14, 2010 5:25 am
Forum: Data Sufficiency
Topic: Bank Assets
Replies: 7
Views: 2164

Sanju09! IMO B Sum of digits of the number 8*10^k + j is equal to 8 + j. The number 8*10^k + j is divisible by 9 without remainder if 8 + j = 9 Remainder is equal to 8 + j - 9 = j - 1 (1) k = 13 Statement (1) ALONE is INSUFFICIENT. We should eliminate answer choices A and D. (2) j = 1 Remainder is e...

by Maciek

Tue Sep 14, 2010 5:06 am
Forum: Data Sufficiency
Topic: when 8 × 10^k + j is divided by 9
Replies: 2
Views: 3090

Sanju09! IMO E 225 = 3*3*5*5 = 3^2*5^2 216 = 2*2*2*3*3*3 = 2^3*3^3 Formula for least common multiple( LCM) of A and B: LCM(A,B) = |A*B|/GCD(A,B) greatest common divisor( GCD) of 225 and 216 is GCD(225,216) = 3*3 = 3^2 LCM(225,216) = |225*216|/GCD(225,216) = |3^2*5^2*2^3*3^3|/3^2 = 2^3*3^3*5^2 k = 2^...

by Maciek

Tue Sep 14, 2010 4:56 am
Forum: Problem Solving
Topic: k = 2^a3^b5^c
Replies: 3
Views: 1575

Hi! k = (x -1) (x +2) - (x -1) (x -2) = x^2 + x - 2 -(x^2 - 3x + 2) = 4x - 4 = 4(x - 1) k = 4(x - 1) (A) if x is ODD then k = 4(ODD - 1) = 4*EVEN = EVEN*EVEN = EVEN This answer choice is incorrect (B) if x is EVEN then k = 4(EVEN - 1) = 4*ODD = EVEN*ODD = EVEN This answer choice is incorrect (C) if ...

by Maciek

Tue Sep 14, 2010 4:37 am
Forum: Problem Solving
Topic: and k = (x –1) (x +2) – (x –1) (x –2)
Replies: 3
Views: 1063

Singhsa!

You will find the solution here:
http://www.beatthegmat.com/at-least-one ... tml#297230

Hope it helps!
Best,
Maciek

by Maciek

Tue Sep 14, 2010 4:24 am
Forum: Problem Solving
Topic: Exponents
Replies: 1
Views: 1147